Inconsistencia en el potencial delta

Encontré una inconsistencia en el potencial delta unidimensional. Supongamos que tenemos un pozo cuadrado unidimensional infinitamente profundo de L a + L . Sabemos que los estados propios son funciones seno y coseno. Son pares o impares.

Ahora agreguemos un potencial delta gramo d ( X ) en el centro. Aquí gramo R . Por consideración de paridad, los estados impares no se ven afectados en absoluto. Los estados pares están acoplados entre sí. Esto significa que los nuevos estados propios de paridad par son superposiciones lineales de los estados propios de paridad par impar. A su vez, esto significa que los estados propios de paridad par deben tener derivada cero en X = 0 .

Sin embargo, si resuelve este mismo problema de otra forma estándar, es decir, integrando la ecuación de Schrödinger a lo largo del potencial delta, obtendrá una condición límite para la parte derecha de la función de onda en la forma de a ψ ( 0 ) + b ψ ( 0 ) = 0 , dónde a y b son números finitos. Esta condición de frontera significa ψ ( 0 ) 0 . ¡Entonces los dos enfoques producen resultados diferentes! ¿Cómo resolvemos esta inconsistencia?

tengo una duda parecida. Incluso sin el potencial delta, no puede aplicar bien la condición de contorno derivada en 1-d. La derivada es distinta de cero en el límite dentro del pozo y cero fuera. Pero simplemente elegimos ignorar esa condición. ¿Por qué?
No entiendo por qué en el delta esperas ψ ( 0 + ) = b a ψ ( 0 + ) . Y si b y a son números reales, eso implica que ninguna probabilidad cruza la barrera.

Respuestas (1)

El punto principal es que, aunque una serie de modos coseno de Fourier convergente puntualmente es una función par ψ ( X ) = ψ ( X ) , no tiene que ser diferenciable en X = 0 . Una suma infinita convergente puntual de funciones diferenciables no es necesariamente una función diferenciable.

De manera más general, como OP ya menciona, la función de onda ψ ( X ) no es necesariamente diferenciable en el X -posición del potencial delta y las dos paredes del pozo. Se debe permitir una discontinuidad en el ψ ( X ) en estos tres X -posiciones. Ver también, por ejemplo, mi respuesta Phys.SE aquí .

Debe haber alguna forma matemática de evitarlo. Pero, el punto es que ahora ya no me atrevo a usar el potencial delta. Al menos para el problema concreto anterior, no estoy seguro de que los dos enfoques produzcan el mismo resultado.